Gravitational Field of a
 Long Moving Rod

Back to Physics World
Back to General Relativity


Problem: Consider an infinitely long thin rod, with linear proper mass density l0, which lays at rest on the x-axis in frame S0. Neglect all stresses in the rod. In frame S, which is moving in the +x direction with speed v, the rod is moving in the -x direction with speed v. Define r0 such the Newtonian potential vanishes at r0, i.e. F (r0) = 0. Using the weak field limit

(a) Find the gravitational field, gmn, in S by transforming from S0 to S using a background Lorentz transformation.
(b) Find duz /dt for u = 0, r = r0, and y = 0 where uz º dz/dt for a particle in free-fall.
(c) Find dP
z /dt for u º |u| (relativistic speed) = speed of particle. Let and z = r0 for a particle in free-fall.


Solution (a): It can be shown that the gravitational field in S0 is given by

where C is an arbitrary constant and F = c2h0/2. Since the field is weak we employ a background Lorentz transformation , which is a transformation of the form

where

The transformation on the left is a boost in the +x direction with speed v. The transformation on the right is the inverse transformation. Transform the gravitational field using Eq. (2). The non-vanishing components are h00, h01, h11, h22 and h33. Noting that r = r (y, z) remains unchanged since y and z remain unchanged and therefore so does F. We find the components of the field in the new frame

Summary:

The metric in S then becomes


Solution (b): The acceleration in the weak field limit is given by

At r = r0 = z, y = 0, for u = 0, dt/dt = 1. U3 = -uz and U0 = c. Eq. (11) then becomes

The derivative in Eq. (12) is then (Note: y = 0 Þ r = z = r0)

l = g2l0 is the linear mass density as measured in S. Eq. (12), with z = r0 then becomes


Solution (c): Multiply Eq. (11) through by the proper mass, m0, to give

Set m = 3 to find dpx/dt for a particle in free-fall, with z = r0, becomes


Back to General Relativity
Back to Physics World

 

Hosted by www.Geocities.ws

1